Application of the Sylow Theorems to groups of order $p^2q$












24












$begingroup$


I am trying to show that any group of order $p^2q$ has a normal Sylow subgroup where $p$ and $q$ are distinct primes.



In the case $p>q$ I have no problem.. By Sylow $n_p|q$, so $n_p$ is either $1$ or $q$. But we also have $n_pequiv1mod p$ which rules out $q$. So in the case $p>q$, $n_p=1$ and the unique p-Sylow subgroup is normal.



In the case $p<q$ however, I run into a problem.. Again we have $n_q|p^2$ (so $n_qin{1,p,p^2}$). Again, the condition $n_qequiv1mod q$ rules out $p$.
Now I am attempting to rule out the case $n_q=p^2$: Assume $n_q=p^2$. Then $p^2equiv1mod qimplies (p+1)(p-1)equiv0mod qimplies q$ must divide $(p+1)$ since $p<q$ and $q$ prime. But since $p<q$ and $q|(p+1)$ we see $q=p+1$. For primes this only happens when $p=2, q=3$.



Does this mean I need to check groups of order $2^2cdot3=12$ or did I miss something along the way that lets me conclude $n_qneq p^2$ and thus that $n_q=1$.










share|cite|improve this question









$endgroup$

















    24












    $begingroup$


    I am trying to show that any group of order $p^2q$ has a normal Sylow subgroup where $p$ and $q$ are distinct primes.



    In the case $p>q$ I have no problem.. By Sylow $n_p|q$, so $n_p$ is either $1$ or $q$. But we also have $n_pequiv1mod p$ which rules out $q$. So in the case $p>q$, $n_p=1$ and the unique p-Sylow subgroup is normal.



    In the case $p<q$ however, I run into a problem.. Again we have $n_q|p^2$ (so $n_qin{1,p,p^2}$). Again, the condition $n_qequiv1mod q$ rules out $p$.
    Now I am attempting to rule out the case $n_q=p^2$: Assume $n_q=p^2$. Then $p^2equiv1mod qimplies (p+1)(p-1)equiv0mod qimplies q$ must divide $(p+1)$ since $p<q$ and $q$ prime. But since $p<q$ and $q|(p+1)$ we see $q=p+1$. For primes this only happens when $p=2, q=3$.



    Does this mean I need to check groups of order $2^2cdot3=12$ or did I miss something along the way that lets me conclude $n_qneq p^2$ and thus that $n_q=1$.










    share|cite|improve this question









    $endgroup$















      24












      24








      24


      6



      $begingroup$


      I am trying to show that any group of order $p^2q$ has a normal Sylow subgroup where $p$ and $q$ are distinct primes.



      In the case $p>q$ I have no problem.. By Sylow $n_p|q$, so $n_p$ is either $1$ or $q$. But we also have $n_pequiv1mod p$ which rules out $q$. So in the case $p>q$, $n_p=1$ and the unique p-Sylow subgroup is normal.



      In the case $p<q$ however, I run into a problem.. Again we have $n_q|p^2$ (so $n_qin{1,p,p^2}$). Again, the condition $n_qequiv1mod q$ rules out $p$.
      Now I am attempting to rule out the case $n_q=p^2$: Assume $n_q=p^2$. Then $p^2equiv1mod qimplies (p+1)(p-1)equiv0mod qimplies q$ must divide $(p+1)$ since $p<q$ and $q$ prime. But since $p<q$ and $q|(p+1)$ we see $q=p+1$. For primes this only happens when $p=2, q=3$.



      Does this mean I need to check groups of order $2^2cdot3=12$ or did I miss something along the way that lets me conclude $n_qneq p^2$ and thus that $n_q=1$.










      share|cite|improve this question









      $endgroup$




      I am trying to show that any group of order $p^2q$ has a normal Sylow subgroup where $p$ and $q$ are distinct primes.



      In the case $p>q$ I have no problem.. By Sylow $n_p|q$, so $n_p$ is either $1$ or $q$. But we also have $n_pequiv1mod p$ which rules out $q$. So in the case $p>q$, $n_p=1$ and the unique p-Sylow subgroup is normal.



      In the case $p<q$ however, I run into a problem.. Again we have $n_q|p^2$ (so $n_qin{1,p,p^2}$). Again, the condition $n_qequiv1mod q$ rules out $p$.
      Now I am attempting to rule out the case $n_q=p^2$: Assume $n_q=p^2$. Then $p^2equiv1mod qimplies (p+1)(p-1)equiv0mod qimplies q$ must divide $(p+1)$ since $p<q$ and $q$ prime. But since $p<q$ and $q|(p+1)$ we see $q=p+1$. For primes this only happens when $p=2, q=3$.



      Does this mean I need to check groups of order $2^2cdot3=12$ or did I miss something along the way that lets me conclude $n_qneq p^2$ and thus that $n_q=1$.







      abstract-algebra group-theory finite-groups






      share|cite|improve this question













      share|cite|improve this question











      share|cite|improve this question




      share|cite|improve this question










      asked Aug 16 '11 at 23:35









      RHPRHP

      1,42311525




      1,42311525






















          3 Answers
          3






          active

          oldest

          votes


















          24












          $begingroup$

          Here's another approach: suppose $n_q=p^2$. The $p^2$ $q$-Sylows each have $q-1$ nonidentity elements, and since a group of prime order is generated by any of its nonidentity elements, these sets are disjoint. So our group has $p^2(q-1)$ elements of order $q$, and so only $p^2$ elements of any other order. There's thus only room for one $p$-Sylow, which is thus normal.



          This possibiity does in fact occur: $A_4$ has order 12, has the Klein group as a normal $2$-Sylow, and all $8$ other elements are $3$-cycles.






          share|cite|improve this answer









          $endgroup$





















            10












            $begingroup$

            Everything you've written seems to be correct. But you don't necessarily have to know the groups of order $12$.



            To finish questions like this one off, it's often helpful to count elements. Here, if a group of order $12$ has more than one Sylow $3$-subgroup then there are four, and these intersect trivially. If you remove the elements of period $3$ from the group, then you aren't left with much!






            share|cite|improve this answer











            $endgroup$





















              2












              $begingroup$

              Consider the case of $q > p$. Then,



              $n_q = p^2$ or $1$ and



              $n_p = q$ or $1$.



              Let G have no non-trivial normal subgroup. Then,



              $n_q = p^2$ and $n_p=q$. Then there are $p^2 (q-1) $ elements of order $q$ and $q(p-1) $ elements of order $p$. So, we have the following ,
              begin{align}
              p^2q &geq p^2(q-1) + q(p^2-1) + 1 \
              & geq p^2q+p^2q-p^2-q+1 \
              & geq p^2q + (p^2-1) (q-1)
              end{align}

              $$implies (p^2-1)(q-1) = 0 implies p=1,-1 text{ or } q=1$$
              But $p$ and $q$ are primes.



              Hence $G$ has a non-trivial normal subgroup.






              share|cite|improve this answer











              $endgroup$













                Your Answer





                StackExchange.ifUsing("editor", function () {
                return StackExchange.using("mathjaxEditing", function () {
                StackExchange.MarkdownEditor.creationCallbacks.add(function (editor, postfix) {
                StackExchange.mathjaxEditing.prepareWmdForMathJax(editor, postfix, [["$", "$"], ["\\(","\\)"]]);
                });
                });
                }, "mathjax-editing");

                StackExchange.ready(function() {
                var channelOptions = {
                tags: "".split(" "),
                id: "69"
                };
                initTagRenderer("".split(" "), "".split(" "), channelOptions);

                StackExchange.using("externalEditor", function() {
                // Have to fire editor after snippets, if snippets enabled
                if (StackExchange.settings.snippets.snippetsEnabled) {
                StackExchange.using("snippets", function() {
                createEditor();
                });
                }
                else {
                createEditor();
                }
                });

                function createEditor() {
                StackExchange.prepareEditor({
                heartbeatType: 'answer',
                autoActivateHeartbeat: false,
                convertImagesToLinks: true,
                noModals: true,
                showLowRepImageUploadWarning: true,
                reputationToPostImages: 10,
                bindNavPrevention: true,
                postfix: "",
                imageUploader: {
                brandingHtml: "Powered by u003ca class="icon-imgur-white" href="https://imgur.com/"u003eu003c/au003e",
                contentPolicyHtml: "User contributions licensed under u003ca href="https://creativecommons.org/licenses/by-sa/3.0/"u003ecc by-sa 3.0 with attribution requiredu003c/au003e u003ca href="https://stackoverflow.com/legal/content-policy"u003e(content policy)u003c/au003e",
                allowUrls: true
                },
                noCode: true, onDemand: true,
                discardSelector: ".discard-answer"
                ,immediatelyShowMarkdownHelp:true
                });


                }
                });














                draft saved

                draft discarded


















                StackExchange.ready(
                function () {
                StackExchange.openid.initPostLogin('.new-post-login', 'https%3a%2f%2fmath.stackexchange.com%2fquestions%2f57938%2fapplication-of-the-sylow-theorems-to-groups-of-order-p2q%23new-answer', 'question_page');
                }
                );

                Post as a guest















                Required, but never shown

























                3 Answers
                3






                active

                oldest

                votes








                3 Answers
                3






                active

                oldest

                votes









                active

                oldest

                votes






                active

                oldest

                votes









                24












                $begingroup$

                Here's another approach: suppose $n_q=p^2$. The $p^2$ $q$-Sylows each have $q-1$ nonidentity elements, and since a group of prime order is generated by any of its nonidentity elements, these sets are disjoint. So our group has $p^2(q-1)$ elements of order $q$, and so only $p^2$ elements of any other order. There's thus only room for one $p$-Sylow, which is thus normal.



                This possibiity does in fact occur: $A_4$ has order 12, has the Klein group as a normal $2$-Sylow, and all $8$ other elements are $3$-cycles.






                share|cite|improve this answer









                $endgroup$


















                  24












                  $begingroup$

                  Here's another approach: suppose $n_q=p^2$. The $p^2$ $q$-Sylows each have $q-1$ nonidentity elements, and since a group of prime order is generated by any of its nonidentity elements, these sets are disjoint. So our group has $p^2(q-1)$ elements of order $q$, and so only $p^2$ elements of any other order. There's thus only room for one $p$-Sylow, which is thus normal.



                  This possibiity does in fact occur: $A_4$ has order 12, has the Klein group as a normal $2$-Sylow, and all $8$ other elements are $3$-cycles.






                  share|cite|improve this answer









                  $endgroup$
















                    24












                    24








                    24





                    $begingroup$

                    Here's another approach: suppose $n_q=p^2$. The $p^2$ $q$-Sylows each have $q-1$ nonidentity elements, and since a group of prime order is generated by any of its nonidentity elements, these sets are disjoint. So our group has $p^2(q-1)$ elements of order $q$, and so only $p^2$ elements of any other order. There's thus only room for one $p$-Sylow, which is thus normal.



                    This possibiity does in fact occur: $A_4$ has order 12, has the Klein group as a normal $2$-Sylow, and all $8$ other elements are $3$-cycles.






                    share|cite|improve this answer









                    $endgroup$



                    Here's another approach: suppose $n_q=p^2$. The $p^2$ $q$-Sylows each have $q-1$ nonidentity elements, and since a group of prime order is generated by any of its nonidentity elements, these sets are disjoint. So our group has $p^2(q-1)$ elements of order $q$, and so only $p^2$ elements of any other order. There's thus only room for one $p$-Sylow, which is thus normal.



                    This possibiity does in fact occur: $A_4$ has order 12, has the Klein group as a normal $2$-Sylow, and all $8$ other elements are $3$-cycles.







                    share|cite|improve this answer












                    share|cite|improve this answer



                    share|cite|improve this answer










                    answered Aug 16 '11 at 23:56









                    Chris EagleChris Eagle

                    29.1k26998




                    29.1k26998























                        10












                        $begingroup$

                        Everything you've written seems to be correct. But you don't necessarily have to know the groups of order $12$.



                        To finish questions like this one off, it's often helpful to count elements. Here, if a group of order $12$ has more than one Sylow $3$-subgroup then there are four, and these intersect trivially. If you remove the elements of period $3$ from the group, then you aren't left with much!






                        share|cite|improve this answer











                        $endgroup$


















                          10












                          $begingroup$

                          Everything you've written seems to be correct. But you don't necessarily have to know the groups of order $12$.



                          To finish questions like this one off, it's often helpful to count elements. Here, if a group of order $12$ has more than one Sylow $3$-subgroup then there are four, and these intersect trivially. If you remove the elements of period $3$ from the group, then you aren't left with much!






                          share|cite|improve this answer











                          $endgroup$
















                            10












                            10








                            10





                            $begingroup$

                            Everything you've written seems to be correct. But you don't necessarily have to know the groups of order $12$.



                            To finish questions like this one off, it's often helpful to count elements. Here, if a group of order $12$ has more than one Sylow $3$-subgroup then there are four, and these intersect trivially. If you remove the elements of period $3$ from the group, then you aren't left with much!






                            share|cite|improve this answer











                            $endgroup$



                            Everything you've written seems to be correct. But you don't necessarily have to know the groups of order $12$.



                            To finish questions like this one off, it's often helpful to count elements. Here, if a group of order $12$ has more than one Sylow $3$-subgroup then there are four, and these intersect trivially. If you remove the elements of period $3$ from the group, then you aren't left with much!







                            share|cite|improve this answer














                            share|cite|improve this answer



                            share|cite|improve this answer








                            edited Aug 17 '11 at 0:06

























                            answered Aug 16 '11 at 23:55









                            Dylan MorelandDylan Moreland

                            16.9k23564




                            16.9k23564























                                2












                                $begingroup$

                                Consider the case of $q > p$. Then,



                                $n_q = p^2$ or $1$ and



                                $n_p = q$ or $1$.



                                Let G have no non-trivial normal subgroup. Then,



                                $n_q = p^2$ and $n_p=q$. Then there are $p^2 (q-1) $ elements of order $q$ and $q(p-1) $ elements of order $p$. So, we have the following ,
                                begin{align}
                                p^2q &geq p^2(q-1) + q(p^2-1) + 1 \
                                & geq p^2q+p^2q-p^2-q+1 \
                                & geq p^2q + (p^2-1) (q-1)
                                end{align}

                                $$implies (p^2-1)(q-1) = 0 implies p=1,-1 text{ or } q=1$$
                                But $p$ and $q$ are primes.



                                Hence $G$ has a non-trivial normal subgroup.






                                share|cite|improve this answer











                                $endgroup$


















                                  2












                                  $begingroup$

                                  Consider the case of $q > p$. Then,



                                  $n_q = p^2$ or $1$ and



                                  $n_p = q$ or $1$.



                                  Let G have no non-trivial normal subgroup. Then,



                                  $n_q = p^2$ and $n_p=q$. Then there are $p^2 (q-1) $ elements of order $q$ and $q(p-1) $ elements of order $p$. So, we have the following ,
                                  begin{align}
                                  p^2q &geq p^2(q-1) + q(p^2-1) + 1 \
                                  & geq p^2q+p^2q-p^2-q+1 \
                                  & geq p^2q + (p^2-1) (q-1)
                                  end{align}

                                  $$implies (p^2-1)(q-1) = 0 implies p=1,-1 text{ or } q=1$$
                                  But $p$ and $q$ are primes.



                                  Hence $G$ has a non-trivial normal subgroup.






                                  share|cite|improve this answer











                                  $endgroup$
















                                    2












                                    2








                                    2





                                    $begingroup$

                                    Consider the case of $q > p$. Then,



                                    $n_q = p^2$ or $1$ and



                                    $n_p = q$ or $1$.



                                    Let G have no non-trivial normal subgroup. Then,



                                    $n_q = p^2$ and $n_p=q$. Then there are $p^2 (q-1) $ elements of order $q$ and $q(p-1) $ elements of order $p$. So, we have the following ,
                                    begin{align}
                                    p^2q &geq p^2(q-1) + q(p^2-1) + 1 \
                                    & geq p^2q+p^2q-p^2-q+1 \
                                    & geq p^2q + (p^2-1) (q-1)
                                    end{align}

                                    $$implies (p^2-1)(q-1) = 0 implies p=1,-1 text{ or } q=1$$
                                    But $p$ and $q$ are primes.



                                    Hence $G$ has a non-trivial normal subgroup.






                                    share|cite|improve this answer











                                    $endgroup$



                                    Consider the case of $q > p$. Then,



                                    $n_q = p^2$ or $1$ and



                                    $n_p = q$ or $1$.



                                    Let G have no non-trivial normal subgroup. Then,



                                    $n_q = p^2$ and $n_p=q$. Then there are $p^2 (q-1) $ elements of order $q$ and $q(p-1) $ elements of order $p$. So, we have the following ,
                                    begin{align}
                                    p^2q &geq p^2(q-1) + q(p^2-1) + 1 \
                                    & geq p^2q+p^2q-p^2-q+1 \
                                    & geq p^2q + (p^2-1) (q-1)
                                    end{align}

                                    $$implies (p^2-1)(q-1) = 0 implies p=1,-1 text{ or } q=1$$
                                    But $p$ and $q$ are primes.



                                    Hence $G$ has a non-trivial normal subgroup.







                                    share|cite|improve this answer














                                    share|cite|improve this answer



                                    share|cite|improve this answer








                                    edited Jan 21 at 7:14

























                                    answered Jan 8 at 7:08









                                    pavanpavan

                                    914




                                    914






























                                        draft saved

                                        draft discarded




















































                                        Thanks for contributing an answer to Mathematics Stack Exchange!


                                        • Please be sure to answer the question. Provide details and share your research!

                                        But avoid



                                        • Asking for help, clarification, or responding to other answers.

                                        • Making statements based on opinion; back them up with references or personal experience.


                                        Use MathJax to format equations. MathJax reference.


                                        To learn more, see our tips on writing great answers.




                                        draft saved


                                        draft discarded














                                        StackExchange.ready(
                                        function () {
                                        StackExchange.openid.initPostLogin('.new-post-login', 'https%3a%2f%2fmath.stackexchange.com%2fquestions%2f57938%2fapplication-of-the-sylow-theorems-to-groups-of-order-p2q%23new-answer', 'question_page');
                                        }
                                        );

                                        Post as a guest















                                        Required, but never shown





















































                                        Required, but never shown














                                        Required, but never shown












                                        Required, but never shown







                                        Required, but never shown

































                                        Required, but never shown














                                        Required, but never shown












                                        Required, but never shown







                                        Required, but never shown







                                        Popular posts from this blog

                                        Can a sorcerer learn a 5th-level spell early by creating spell slots using the Font of Magic feature?

                                        Does disintegrating a polymorphed enemy still kill it after the 2018 errata?

                                        A Topological Invariant for $pi_3(U(n))$